1answer.
Ask question
Login Signup
Ask question
All categories
  • English
  • Mathematics
  • Social Studies
  • Business
  • History
  • Health
  • Geography
  • Biology
  • Physics
  • Chemistry
  • Computers and Technology
  • Arts
  • World Languages
  • Spanish
  • French
  • German
  • Advanced Placement (AP)
  • SAT
  • Medicine
  • Law
  • Engineering
kipiarov [429]
3 years ago
12

I recently (yesterday) took the AMC 10A, can anyone help me with this problem?

Mathematics
1 answer:
lianna [129]3 years ago
8 0
First, we claim that \frac{10^n-1}{9} is the n-digit number with all digits equal to one.

Note that 10^n is a one followed by n zeroes, so subtracting one gives n nines.  Divide that number by nine and you get n ones, completing the proof.

Therefore, we have that A_n = a \frac{10^n - 1}{9}, B_n = b \frac{10^n-1}{9}, and C_n = c \frac{10^{2n} - 1}{9}.

Let x = 10^n.  Then, we have:

c \frac{x^2-1}{9} - b \frac{x-1}{9} = (a \frac{10^n - 1}{9})^2 = a^2 \frac{x^2 - 2x + 1}{81}.

Multiplying by 81 gives:

9c(x^2-1) - 9b(x-1) = a^2 (x^2-2x+1)

Now, note that x=1 is not a valid input, since 10^n = 1 requires n=0, so we safely divide by x-1 to get:

9c(x+1) - 9b = a^2(x-1)

9cx + 9c - 9b = a^2x - a^2

x(9c-a^2) = 9b-9c-a^2

Because this is now a linear equation in x, it has either zero, one, or infinitely many solutions.  Obviously, we need the latter to occur, which happens when 9c=a^2 and 9b-9c-a^2 = 0, since the coefficient of x must cancel to zero and thus the RHS must equal zero as well.

Since 9c = a^2, we must have a = 3 \sqrt{c}.  Since a must be a multiple of three, we plug in values.  If a = 9, we get c = 9 and thus 9b - 162 = 0, which is impossible.  So a = 9 doesn't work.

With a = 6, c = 4, and thus 9b-72=0, so 9b=72, so b=8.  This gives 6+8+4=18 as one possibility.

With a = 3, c = 1, and thus 9b - 18 = 0, so b = 2.  This obviously is worse.

We've gone through all the cases and the two possibilities are 2 and 18, so our answer is \boxed{18}.


You might be interested in
3 Meg found another treasure between -14 feet and 1 -22 feet. Where was that treasure?​
IceJOKER [234]

Answer:

- 18 feet

Step-by-step explanation:

3 0
3 years ago
Seed mixture X is 40 percent ryegrass and 60 percent bluegrass by weight; seed mixture Y is 25 percent ryegrass and 75.percent f
irinina [24]

Answer:

33%

Step-by-step explanation:

Assuming the weight of the mixture to be 100g**, then the weight of ryegrass in the mixture would be 30g.

Also, assume the weight mixture X used in the mixture is Xg, then the weight of mixture Y used in the mixture would be (100-X)g.

So we can now equate the parts of the ryegrass in the mixture as:

0.4X + 0.25(100-X) = 30

<=> 0.4X + 25 - 0.25X = 30

<=> 0.15X = 5

<=> X = 5/0.15 = 500/15 = 100/3

So the weight of mixture X as a percentage of the weight of the mixture

= (weight of X/weight of mixture) * 100%

= (100/3)/100 * 100%

= 33%

3 0
3 years ago
Answer to this question please?
egoroff_w [7]
7 mphs faster...............
7 0
3 years ago
If there are 680 calories in 8 oz of ground beef, how many
irina [24]

Answer:

3/8ths of 680 kcal or 3 x 85 = 255 kcal.

Step-by-step explanation:

3/8ths of 680 kcal or 3 x 85 = 255 kcal.

6 0
2 years ago
Read 2 more answers
Round 496.042 to the nearest hundredth. Enter your answer in the response box.<br> HElppp
cluponka [151]
124.59 I think not sure
4 0
3 years ago
Other questions:
  • A cable on a crane measures 89 yards 2 feet. How much total cable would be on 14 identical cranes? Convert the answer to larger
    11·2 answers
  • Can somebody plz answer all these questions that would be amazing!!!!
    7·1 answer
  • How does a digit in the ten thousands place compare to a digit in the thousands place
    5·1 answer
  • It is $3.00 an hour to rent a skateboard and $4.00 to rent safety equipment. You are required to wear safety equipment in the Sk
    13·1 answer
  • paid after the grace period, on average, more than 4 times in 2018, what are the null and alternative hypotheses? Select the cor
    12·1 answer
  • The chart gives three different international calling plans. Jamie has budgeted $20 per month for international calls. Round ans
    11·1 answer
  • A pizza company uses equation C = 15n to calculate the cost of buying a number of pizzas. What is the constant of proportionalit
    14·2 answers
  • Find the area of the
    11·1 answer
  • Solve for x: 9x-1=10x+9<br><br> How do you get x off one side and onto the other, please explain
    15·1 answer
  • HELP MEEE???? L'LL GIVE U A BRAINLIST???
    5·1 answer
Add answer
Login
Not registered? Fast signup
Signup
Login Signup
Ask question!